Difference between revisions of "2011 AMC 12B Problems/Problem 17"
(→Solution 2 (Non-Rigorous Trends)) |
m (Added parentheses) |
||
(4 intermediate revisions by 2 users not shown) | |||
Line 8: | Line 8: | ||
<math>g(x)=\log_{10}\left(\frac{x}{10}\right)=\log_{10}\left({x}\right) - 1</math> | <math>g(x)=\log_{10}\left(\frac{x}{10}\right)=\log_{10}\left({x}\right) - 1</math> | ||
− | <math>h_{1}(x)=g(f(x))\text{ = }g(10^{10x}=\log_{10}\left({10^{10x}}\right){ - 1 = 10x - 1}</math> | + | <math>h_{1}(x)=g(f(x))\text{ = }g(10^{10x})=\log_{10}\left({10^{10x}}\right){ - 1 = 10x - 1}</math> |
Proof by induction that <math>h_{n}(x)\text{ = }10^n x - (1 + 10 + 10^2 + ... + 10^{n-1})</math>: | Proof by induction that <math>h_{n}(x)\text{ = }10^n x - (1 + 10 + 10^2 + ... + 10^{n-1})</math>: | ||
Line 30: | Line 30: | ||
The sum of the digits is 8 times 2010 plus 9, or <math>\boxed{16089\textbf{(B)}}</math> | The sum of the digits is 8 times 2010 plus 9, or <math>\boxed{16089\textbf{(B)}}</math> | ||
− | ==Solution 2 (Non-Rigorous Trends)== | + | ==Solution 2 (Quick, Non-Rigorous Trends)== |
− | As before, <math>h_1(x)=10x-1</math>. Compute <math>h_1</math>, <math>h_2</math>, and <math>h_3 | + | As before, <math>h_1(x)=10x-1</math>. Compute <math>h_1(x)</math>, <math>h_2(x)</math>, and <math>h_3(x)</math> to yield 9, 89, and 889. Notice how this trend will repeat this trend (multiply by 10, subtract 1, repeat). As such, <math>h_{2011}</math> is just 2010 8's followed by a nine. <math>2010(8)+9=\boxed{\textbf{B)}16089}</math>. |
~~BJHHar | ~~BJHHar |
Latest revision as of 15:40, 21 September 2024
Problem
Let , and for integers . What is the sum of the digits of ?
Solution
Proof by induction that :
For ,
Assume is true for n:
Therefore, if it is true for n, then it is true for n+1; since it is also true for n = 1, it is true for all positive integers n.
, which is the 2011-digit number 8888...8889
The sum of the digits is 8 times 2010 plus 9, or
Solution 2 (Quick, Non-Rigorous Trends)
As before, . Compute , , and to yield 9, 89, and 889. Notice how this trend will repeat this trend (multiply by 10, subtract 1, repeat). As such, is just 2010 8's followed by a nine. .
~~BJHHar
See also
2011 AMC 12B (Problems • Answer Key • Resources) | |
Preceded by Problem 16 |
Followed by Problem 18 |
1 • 2 • 3 • 4 • 5 • 6 • 7 • 8 • 9 • 10 • 11 • 12 • 13 • 14 • 15 • 16 • 17 • 18 • 19 • 20 • 21 • 22 • 23 • 24 • 25 | |
All AMC 12 Problems and Solutions |
The problems on this page are copyrighted by the Mathematical Association of America's American Mathematics Competitions.